Talk:2015 AMC 10A Problems/Problem 25

Revision as of 13:03, 5 February 2015 by F.11235 (talk | contribs) (Created page with "The posted solution relies on calculus, and is not appropriate for the AMC contest. Please update to reflect a geometric probability solution (or borrow the one from nsun48 in th...")
(diff) ← Older revision | Latest revision (diff) | Newer revision → (diff)

The posted solution relies on calculus, and is not appropriate for the AMC contest. Please update to reflect a geometric probability solution (or borrow the one from nsun48 in the discussion of this problem at http://www.artofproblemsolving.com/Forum/viewtopic.php?f=133&t=623881)